une série

bsr,
une serie m'empeche de dormir $\sum \frac{1}{2^n \mid sin(n)\mid}$
merci

Réponses

  • Que veux tu faire de cette série? Montrer sa convergence ou la calculer?
  • seulement etudier sa nature
  • Essaie de majorer ta somme, rappelle toi du théorème de comparaison...
  • tu as une majoration!!
  • 0< |sin n| < 1

    D'où SOMME ( 1/[(2^n).|sin n|] ) < SOMME ( 1/(2^n) )

    quelque soit n appartenant à IN

    A toi de conclure.
  • Et non <SPAN CLASS="MATH"><IMG WIDTH="73" HEIGHT="32" ALIGN="MIDDLE" BORDER="0" SRC="http://www.les-mathematiques.net/phorum/2006/11/25/102507/cv/img1.png&quot; ALT="$ \vert sin n\vert<1$"></SPAN> n'implique pas <SPAN CLASS="MATH"><IMG WIDTH="134" HEIGHT="35" ALIGN="MIDDLE" BORDER="0" SRC="http://www.les-mathematiques.net/phorum/2006/11/25/102507/cv/img2.png&quot; ALT="$ \frac{1}{2^n\vert sin(n)\vert}\leq frac{1}{2}$"></SPAN> mais l'inégalité dans l'autre sens qui n'est pas utile du coup.
    <BR>
    <BR>Mais une petite transfo d'Abel résout tout ça non ? Après il y a sûrement plus simple
  • tu as une minoration et pas une majoration!!
  • La transfo d'Abel marche si le sinus est au numérateur.
    En fait je me demande d'abord si le terme général tend vers 0 facilement
  • la mesure d'irrationalité est inférieure à $9$ (théorème de Hata), donc
    $|\frac{p}{q} - \pi| > \frac{C}{q^9}$ pour une certaine constante $C$ et pour $q$ assez grand (plus grand qu'une constante $q_0$). Cela montre que $| \sin(n)| \geq \frac{K}{n^8}$ pour $n$ assez grand et une constante $K>0$. Cela permet de déduire la convergence de la série .. quelqu'un avec une solution plus élémentaire ?
  • la mesure d'irrationalité de $\pi$ est inférieure à $9$ (théorème de Hata), donc
    $|\frac{p}{q} - \pi| > \frac{C}{q^9}$ pour une certaine constante $C$ et pour $q$ assez grand (plus grand qu'une constante $q_0$). Cela montre que $| \sin(n)| \geq \frac{K}{n^8}$ pour $n$ assez grand et une constante $K>0$. Cela permet de déduire la convergence de la série .. quelqu'un avec une solution plus élémentaire ?
  • avec ce resultat le probleme est resolu mais comment expliquer cela a mon prof!!!! y'a t- il de plus simple
    en faite est ce qu'on peut mesurer l'irrationalité de $\pi $ de facon exacte ou c'est un probleme ouvert!!
  • Bon, la réponse (ou du moins ce qui permet de trouver la réponse) est dans
    " exercices de mathematiques
    ORAUX X-ENS
    analyse 1

    Serge Francinou
    Hervé Gianella
    Serge Nicolas "

    Edition Cassini.

    l'exercice 2.15 page 77 va t'expliquer comment montrer que la série $\sum_{i=0}^{\infty} \frac{1}{nsin(n)}$ est divergente, en utilisant notamment le fait que la suite sin(n) est dense dans [-1;1] (qui est.
    Tu devras aussi passer par le théorème de dirichlet qui ressemble (avec des notions très élémentaires) au théorème utiliser par alekk.

    ensuite une minoration triviale de ta série par la série divergente te résoud ton affaire...et tu (ou plutot on) pourras enfin dormir :D
  • Bonjours,
    D'après le post de "m" : "une minoration triviale de ta série par la série divergente te résout ton affaire"
    Mais d'après le th proposé par Alekk je montre que la série est majorée par une série convergente.
    Je me trompe !!!


    [Salma : M parle de la série 1/(n.|sin n|) alors qu'Alekk traite la série du post initial 1/(2^n.|sin n|) . AD]
  • Cela signifie quoi "la mesure d'irrationalité" ?
  • tu peux lire cet article sur ce sujet qu'on m'as posté.
    www.espacemath.com/images/sinus3.pdf

    et je viens d'apprendre qu'on ne sait pas la limite de $n\mid sin(n)\mid$
  • $n|\sin(n)|$ ne tend pas vers $+\infty$ mais ne converge pas non plus, bref : ne converge pas c'est un exercice première année de la base d'exercice du site dans lequel nous sommes (à moi que tu ne parles de la série ?)
  • oui, on ne peut pas s'en sortir avec seulement Dirichlet
    $|\frac{p}{q} - \pi| \leq \rac{1}{q^2}$
    pour une infinité d'entiers $p,q$ car pour montrer une convergence, on veut une inégalité dans l'autre sens .. On veut une minoration de $| \sin(n) |$, et je ne vois pas comment s'en sortir sans utiliser des théorèmes puissants ..
  • Merci José,
    J'ai regardé dans la base d'exercices la limite de cette suite. La preuve n'est pas claire . Pour s'en convaincre regarde l'article : <http://www.espacemath.com/images/sinus3.pdf&gt;
  • bonsoir,
    j'ai un probleme sur la version du theoreme de Dirichlet. la version que je connais est la suivante:
    si x est un reel, il existe une infinité d'entiers q relatifs tels que \\
    $|\frac{p}{q} - \pi| \leq \frac{1}{q^2}$ \\ pour un certain p. voir ce lien www.dma.ens.fr/culturemath/maths/pdf/nombres/reseaux.pdf \\
    j'aimerai avoir un lien vers cette version : si x est irrationnel, il existe une infinité d'entiers p et q tels que \\
    $|\frac{p}{q} - \pi| \leq \frac{1}{q^2}$
  • soit x est un irrationnel dans ]0,1[ et $n\in\N^*$
    il exite un entier $p\in\{0,...,n-1 \}$ tel que $|x-p/n|
  • En fait, il semble bien que j'ai dit une bêtise ! je voulais raisonner par contraposer, en disant que si la série en 1/(nsin(n)) je convergait pas, la série en terme absolument ne converge pas non plus ! (forcément car si la série converge absolument, alors elle converge).

    En fait, on peut faire une petite manipulation sur le terme de la série et écrire $\sum_{i=1}^{\infty} \frac{1}{2^nsin(n)} = \sum_{i=1}^{\infty} \frac{n^s}{2^n}\frac{1}{n^ssin(n)}$

    Comme pour un certain s, $(n^ssin(n))^(-1)$ --> 0 et que l'autre terme est celui d'une série convergente $\forall s$, le théorème de comparaison donne en faite la convergence.

    Bon, je vous demande confirmation, car on n'est jamais a l'abris de raconter n'importe quoi ;)
  • on sait qu'il existe un entier s>0 tel que $\dfrac{1}{n^s sin(n)}\longrightarrow 0$
    il existe $N$ tel que pour tout $n>N$ $|\dfrac{1}{n^s sin(n)}|N$ on a $|\dfrac{1}{2^nsin(n)}|\leq \dfrac{n^s}{2^n} \dfrac{1}{n^s| sin(n)|}$
    $|\dfrac{1}{2^nsin(n)}|\leq \dfrac{n^s}{2^n}$

    on a majoré $|\dfrac{1}{2^nsin(n)}|$ par le terme général d'une série convergente
Connectez-vous ou Inscrivez-vous pour répondre.